LSAT and Law School Admissions Forum

Get expert LSAT preparation and law school admissions advice from PowerScore Test Preparation.

 Administrator
PowerScore Staff
  • PowerScore Staff
  • Posts: 8919
  • Joined: Feb 02, 2011
|
#40560
Complete Question Explanation
(The complete setup for this game can be found here: lsat/viewtopic.php?t=15423)

The correct answer choice is (D)

There are several different ways to attack this question. First, during the setup we established that U, X, and Z could not perform in slot one, and that only V, W, and Y could perform in slot one. Answer choice (D) contains V, W, and Y (and no other variables) and ultimately answer choice (D) is correct.

If you had some uncertainty over whether you had made all the inferences available in this game, then you could try a second approach. This approaches uses Not Laws and prior work:
  • Not Laws: By applying the Not Laws on slot one, answer choice (E) can be eliminated because it contains Z.

    Prior Work: In questions #2 and #4, we proved that V could perform first, and thus answer choice (A) can be eliminated since it does not contain V. In questions #3 and #4, we proved that Y could perform first, and thus answer choice (B) can be eliminated since it does not contain Y. Finally, in question #3, we proved that W could perform first, and thus answer choice (C) can be eliminated since it does not contain W. Thus, every answer choice but (D) is eliminated by using this method, which is slightly more time-consuming but also slightly more assured to be correct.


Regardless of your approach, answer choice (D) is the correct answer.

Get the most out of your LSAT Prep Plus subscription.

Analyze and track your performance with our Testing and Analytics Package.